Skip to main content
Log in

Ordinal dominance and risk aversion

  • Research Article
  • Published:
Economic Theory Bulletin Aims and scope Submit manuscript

Abstract

We find that, for sufficiently risk-averse agents, strict dominance by pure or mixed actions coincides with dominance by pure actions in the sense of (Börgers in Econometrica 61(2):423–430, 1993), which, in turn, coincides with the classical notion of strict dominance by pure actions when preferences are asymmetric. Since risk aversion is a cardinal feature, all finite single-agent choice problems with ordinal preferences admit compatible utility functions which are sufficiently risk averse as to achieve equivalence between pure and mixed dominance. This result extends to some infinite environments.

This is a preview of subscription content, log in via an institution to check access.

Access this article

Price excludes VAT (USA)
Tax calculation will be finalised during checkout.

Instant access to the full article PDF.

Fig. 1
Fig. 2

Similar content being viewed by others

Notes

  1. We index \(M_{u}\)-dominance by \(u\) to highlight the fact that it depends on the cardinal information embedded in vNM utility functions. In contrast, \(P\)-dominance only depends on the agent’s ordinal state-contingent preferences over actions.

  2. In general, when indifference is allowed, for an action to be strictly dominated by a pure action implies that it is \(P\)-dominated, which implies in turn that it is weakly dominated by a pure action. In the generic case in which all state-contingent preferences are strict, these three notions of pure dominance coincide.

  3. Our research is in a Bayesian framework, so we use “risk” and “uncertainty” as synonyms. Nevertheless, our intuition is closely related to the work of Klibanoff (2001). He asks under which conditions would an uncertainty-averse agent be willing to choose mixed actions. As it turns out, the trade-off between averaging outcomes (uncertainty) and increasing variance (risk) plays a prominent role.

  4. These results are similar in spirit to Lemma 1 in Chen and Luo (2012), which implies that, in “concave-like” games, an action is \(M_{u}\)-dominated if and only if it is strictly dominated by a pure action. However, their lemma is interesting only for uncountable environments (including mixed extensions of finite environments). In finite or countable environments—like the ones we consider—if an agent has concave-like preferences, then there exists a pure action which \(P\)-dominates every other action.

  5. Lo (2000) extends this result to all models of preferences satisfying Savage’s P3 axiom.

  6. This result can be traced back to a result from statistical decision theory in Wald (1947). In the context of rationalizability, it is established for finite games in Pearce (1984), and for compact action spaces and continuous preferences in Zimper (2005) and Daniëls (2008).

  7. Details are available upon request.

References

  • Bernheim, D.: Rationalizable strategic behavior. Econometrica 52(4), 1007–1028 (1984)

    Article  MATH  MathSciNet  Google Scholar 

  • Bohner, M., Gelles, G.M.: Risk aversion and risk vulnerability in the continuous and discrete case. Decis. Econ. Financ. 35(1), 1–28 (2012)

    Article  MATH  MathSciNet  Google Scholar 

  • Bonanno, G.: A syntactic approach to rationality in games with ordinal payoffs. In: Bonanno, G., Van Der Hoek, W., Wooldridge, M. (eds.) Logic and the foundatons of game and decision theory (LOFT 7), Texts in logic and games, vol. 3, pp 59–85. Amsterdam University Press (2008)

  • Börgers, T.: Pure strategy dominance. Econometrica 61(2), 423–430 (1993)

    Article  MATH  MathSciNet  Google Scholar 

  • Chambers, C.P., Echenique, F., Shmaya, E.: General revealed preference theory, Social Science Working Paper 1332, California Institute of Technology (2010)

  • Chen, Y., Luo, X.: An indistinguishability result on rationalizability under general preferences. Econ. Theory 51(1), 1–12 (2012)

    Article  MathSciNet  Google Scholar 

  • Daniëls, T.: Pure strategy dominance with quasiconcave utility functions. Econ. Bull. 2008(3), 54–61 (2008)

    Google Scholar 

  • Epstein, L.G.: Preference, rationalizability and equilibrium. J. Econ. Theory 73(1), 1–29 (1997)

    Article  MATH  Google Scholar 

  • Klibanoff, P.: Characterizing uncertainty aversion through preference for mixtures. Soc. Choice Welf. 18(2), 289–301 (2001)

    Article  MATH  MathSciNet  Google Scholar 

  • Ledyard, J.O.: The scope of the hypothesis of Bayesian equilibrium. J. Econ. Theory 39(1), 59–82 (1986)

    Article  MATH  MathSciNet  Google Scholar 

  • Lo, K.C.: Rationalizability and the Savage axioms. Econ. Theory 15(3), 727–733 (2000)

    Article  MATH  Google Scholar 

  • Pearce, D.: Rationalizable strategic behavior and the problem of perfection. Econometrica 52(4), 1029–1050 (1984)

    Article  MATH  MathSciNet  Google Scholar 

  • Rockafellar, R.T.: Convex analysis, Princeton landmarks in mathematics and physics, vol. 28, Princeton University Press (1996)

  • Wald, A.: An essentially complete class of admissible decision functions. Ann. Math. Stat. 18(4), 549–555 (1947)

    Article  MATH  MathSciNet  Google Scholar 

  • Weinstein, J.: The dependence of solution concepts on risk attitude, mimeo (2014)

  • Zimper, A.: Equivalence between best responses and undominated strategies: a generalization from finite to compact strategy sets. Econ. Bull. 3(7), 1–6 (2005)

    Google Scholar 

Download references

Acknowledgments

This paper originated from a conjecture by Edward Green. We are thankful for his guidance and support, as well as the useful comments from Lisa Posey, Nail Kashaev, Lidia Kosenkova, Jonathan Weinstein, two anonymous referees, and the attendants of the 2014 Spring Midwest Trade and Theory Conference at IUPUI, and the 25\(\mathrm {th}\) International Game Theory Conference at Stony Brook University. We gratefully acknowledge the Human Capital Foundation, (http://www.hcfoundation.ru/en/) and particularly Andrey P. Vavilov, for research support through the Center for the Study of Auctions, Procurements, and Competition Policy (http://capcp.psu.edu/) at the Pennsylvania State University. All remaining errors are our own.

Author information

Authors and Affiliations

Authors

Corresponding author

Correspondence to Bruno Salcedo.

A Proofs

A Proofs

Proof

(Proposition 1) Fix an action \(a\in A\) and a state \(x\in X\), and let \(u\) and \(v\) be compatible vNM utility functions such that

$$\begin{aligned} \{ \alpha \in \Delta (A) \,|\, u(\alpha ,x) \ge u(a,x)\} \subseteq \{ \alpha \in \Delta (A) \,|\, v(\alpha ,x) \ge v(a,x)\}. \end{aligned}$$
(11)

We want to show that \(\tau _u(a,x)\ge \tau _v(a,x)\). If \(a\) is either \({\mathbin \succcurlyeq }_x\)-maximum or \({\mathbin \succcurlyeq }_x\)-minimum, then \(\tau _u(a,x)=+\infty \) and \(\tau _v(a,x)=+\infty \) by definition, and the result is trivial. Hence, we assume for the rest of the proof that \(\underline{u}(x)<u(a,x)<\bar{u}(x)\).

By Assumption 1, there exists an action \(b\in A\) such that \(u(b,x) = u^-(a,x)\), and, consequently, \(v(b,x) = v^-(a,x)\). Let \((a_m)\) be a sequence of actions such that \(a_m{\mathbin \succ }_x a\) for all \(m\), \(\lim _{m\rightarrow \infty } u(a_m,x) = \bar{u}(x)\), and \(\lim _{m\rightarrow \infty } v(a_m,x) = \bar{v}(x)\). Also, for each \(\theta \in [0,1]\) and each \(m\in {\mathbb {N}}\), let \(\alpha _{m,\theta }\) be the mixed action that plays \(a_m\) with probability \(\theta \), and \(b\) with probability \(1-\theta \). For all such \(m\) we have that \(u(\alpha _{m,1},x) > u(a,x) > u(\alpha _{m,0},x)\). Hence, since expected utility is continuous in the mixing probabilities, there exists \(\theta (m)\in (0,1)\) such that \(u(\alpha _{m,\theta (m)},x)= u(a,x)\). After some simple algebra this implies that:

$$\begin{aligned} \dfrac{u(a,x)-u^-(a,x)}{u(a_m,x) - u(a,x)} = \dfrac{\theta (m)}{1-\theta (m)}. \end{aligned}$$
(12)

By (11), we have that \(v(\alpha _{m,\theta (m)},x) \ge v(a,x)\), which implies that:

$$\begin{aligned} \dfrac{v(a,x)-v^-(a,x)}{v(a_m,x) - v(a,x)} \le \dfrac{\theta (m)}{1-\theta (m)}. \end{aligned}$$
(13)

Using (12) and (13) and taking limits as \(m\) goes to infinity thus yield the desired result

$$\begin{aligned} \tau _v(a,x) \!=\! \lim _{m\rightarrow \infty } \dfrac{v(a,x)-v^-(a,x)}{v(a_m,x) - v(a,x)} \!\le \! \lim _{m\rightarrow \infty } \dfrac{u(a,x)-u^-(a,x)}{u(a_m,x) - u(a,x)} \!=\! \tau _u(a,x).\qquad \end{aligned}$$
(14)

\(\square \)

Proof

(Lemma 1) Let \(\beta = \alpha (W_x(a))\). Since \(u(b,x)\le u^-(a,x)\) for \(b\in W_x(a)\), and \(u(b,x)\le \bar{u}(x)\) for \(b\in B\), it follows that:

$$\begin{aligned} u(\alpha ,x) - u(a,x)&\le \beta \big (u^-(a,x) - u(a,x)\big ) + \big (1-\beta \big ) \big ( \bar{u}(x) - u(a,x)\big ) \nonumber \\&= -\beta \left( \dfrac{u^-(a,x) - u(a,x)}{\bar{u}(x) - u(a,x) } \right) \big (\bar{u}(x) - u(a,x)\big )\nonumber \\&+ \big (1-\beta \big ) \big (\bar{u}(x) - u(a,x)\big ) \nonumber \\&= \big (1-\beta \cdot (\tau _u(a,x)+1)\big ) \big ( \bar{u}(x) - u(a,x)\big ) \le 0. \end{aligned}$$
(15)

\(\square \)

Proof

(Proposition 2) Fix a set \(B\subseteq A\), an action \(a \in A\backslash P(B)\), and a mixture \(\alpha \) with \(\alpha (B\backslash \{a\})=1\). There exists some \(Y\subseteq X\) conditional on which \(a\) is not weakly dominated in \(B\). Assume without loss of generality that for all \(b \in B\setminus \{a\}\) there exists some \(x\in Y\) such that \(b\not \sim _x a\). This implies that for all \(b \in B\backslash \{a\}\) there also exists some \(x\in Y\) such that \(a{\mathbin \succ }_x b\), i.e., \(B\backslash \{a\} \subseteq \cup _{x\in Y} W_x(a)\). Since \(K = \min \{\Vert A\Vert ,\Vert X\Vert \}<+\infty \), there exists a finite subset \(Z = \{x_1,\ldots ,x_k\}\subseteq Y\) with cardinality \(k\le K\), and such that \(B\backslash \{a\} \subseteq \cup _{x\in Z} W_x(a)\). Therefore:

$$\begin{aligned} \sum _{x\in Z} \alpha (W_x(a)) \ge \alpha (B\backslash \{a\}) = 1 \ge \dfrac{k}{K} = \sum _{x\in Z} \dfrac{1}{K} \ge \sum _{x\in Z} \dfrac{1}{\tau _u(a,x) + 1}. \end{aligned}$$
(16)

This implies that there exists a state \(x\) such that \((\tau _u(a,x) + 1)\alpha (W_x(a))\ge 1\), and the result thus follows from Lemma 1. \(\square \)

Proof

(Proposition 3) Let \(a\), \(B\), \(\alpha ,\) and \(Y\) be as in the proof of Proposition 2. As before, we know that \(B\setminus \{a\} \subseteq \cup _{x\in Y} W_x(a,B)\), and thus:

$$\begin{aligned} \sum _{x\in Y} \alpha (W_x(a,B)) \ge 1 \ge \sum _{x\in X} \dfrac{1}{1+\tau _u(a,x)} \ge \sum _{x\in Y} \dfrac{1}{1+\tau _u(a,x)}. \end{aligned}$$
(17)

Hence, there exists \(x\in Y\) such that \((\tau _u(a,x) + 1)\alpha (W_x(a))\ge 1\), and the result follows from Lemma 1. \(\square \)

Rights and permissions

Reprints and permissions

About this article

Check for updates. Verify currency and authenticity via CrossMark

Cite this article

Gafarov, B., Salcedo, B. Ordinal dominance and risk aversion. Econ Theory Bull 3, 287–298 (2015). https://doi.org/10.1007/s40505-014-0059-z

Download citation

  • Received:

  • Accepted:

  • Published:

  • Issue Date:

  • DOI: https://doi.org/10.1007/s40505-014-0059-z

Keywords

JEL Classification

Navigation